Robbins and Cotran Review of Pathology Block 1 CH 1-5

Lakukan tugas rumah & ujian kamu dengan baik sekarang menggunakan Quizwiz!

20. A 5-year-old child ingests 50 iron tablets, each with 27 mg of iron. Within 6 hours the child develops abdominal pain and lethargy. On physical examination he is hypotensive. Lab- oratory studies show metabolic acidosis. Through formation of which of the following compounds is the cell injury in this child most likely mediated? A Ascorbic acid B Hemosiderin C Hydroxyl radical D Nitric oxide E Superoxide dismutase

20. C Excessive iron ingestion, particularly by a child, can overwhelm the body's ability to bind the absorbed free iron with the transport protein transferrin. The free iron contributes to generation of cellular free radicals via the Fenton reaction. Ascorbic acid (vitamin C) and vitamin E both act as antioxidants to protect against free radical injury, albeit over a long time frame. Hemosiderin is a storage form of iron from excess local or systemic accumulation of ferritin, and by itself does not cause cell injury until large amounts are present, as with hemochromatosis. Nitric oxide generated within macrophages can be to kill microbes. It can be converted to a highly reactive peroxynitrite anion. Superoxide dismutase helps break down superoxide anion to hydrogen peroxide, thus scavenging free radicals.

20. Patients with extensive endothelial injury from Esch- erichia coli sepsis have consumption of coagulation factors as well as an extensive inflammatory response. Administration of activated protein C is most likely to decrease this inflammatory response by reducing the amount of which of the following substances? A Complement B Fibrin C Kallikrein D Plasmin E Thrombin

20. E Ongoing activation of coagulation generates an inflammatory response that further amplifies coagulation, creating a vicious cycle. Protein C antagonizes coagulation factor V, which catalyzes activation of prothrombin to thrombin, thereby breaking the cycle of thrombin generation. Complement components can become activated by plasmin (C3) and kallikrein (C5), forming anaphylatoxins (C3a and C5a) that promote inflammation. Fibrin, the end product of coagulation pathways, forms a meshwork entrapping platelets and creating a plug. Kallikrein is generated by activation of Hageman factor (XII) and leads to formation of bradykinin. Plasmin is generated from plasminogen activated by thrombosis to pro- mote clot lysis.

21. A 63-year-old man has a 2-year history of worsening congestive heart failure. An echocardiogram shows mitral valve stenosis with left atrial dilation. A mural thrombus is present in the left atrium. One month later, he experiences left flank pain and notes hematuria. Laboratory testing shows an elevated serum AST. The representative microscopic appearance of the lesion is shown in the figure. Which of the following patterns of tissue necrosis is most likely to be present in this man? A Caseous B Coagulative C Fat D Gangrenous E Liquefactive

21. B Embolization of the thrombus led to blockage of a renal arterial branch, causing an acute renal infarction in this patient. An ischemic injury to most internal organs produces a pattern of cell death called coagulative necrosis. Note the faint outlines of renal tubules and glomerulus in the figure, but no cellular nuclei. Caseous necrosis can be seen in various forms of granulomatous inflammation, typified by tuberculosis. Fat necrosis is usually seen in pancreatic and breast tissue. Gangrenous necrosis is a form of coagulative necrosis that usually results from ischemia and affects limbs. Liquefactive necrosis occurs after ischemic injury to the brain and is the pattern seen with abscess formation.

21. A 95-year-old woman touches a pot of boiling water. Within 2 hours, she has marked erythema of the skin of the fingers of her hand, and small blisters appear on the finger pads. This has led to which one of the following inflammatory responses? A Fibrinous inflammation B Granulomatous inflammation C Purulent inflammation D Serous inflammation E Ulceration

21. D Serous inflammation is the mildest form of acute inflammation. A blister is a good example of serous inflammation. It is associated primarily with exudation of fluid into the subcorneal or subepidermal space. Because the injury is mild, the fluid is relatively protein-poor. A protein-rich exudate results in fibrin accumulation. Granulomatous inflammation is characterized by collections of transformed macrophages called epithelioid cells. Acute inflammatory cells, mainly neutrophils, exuded into a body cavity or space form a purulent (suppurative) exudate, typically associated with liquefactive necrosis. Loss of the epithelium leads to ulceration.

1. A 77-year-old woman has chronic renal failure. Her serum urea nitrogen is 40 mg/dL. She is given a diuretic medication and loses 2 kg (4.4 lb). She reduces the protein in her diet and her serum urea nitrogen decreases to 30 mg/dL. Which of the following terms best describes cellular responses to disease and treatment in this woman? A Adaptation B Apoptosis C Necroptosis D Irreversible injury E Metabolic derangement

1. A Normal cells handle physiologic demands and maintain metabolic functions within narrow ranges, termed homeostasis. Under disease conditions with stress on cells, there is adaptation to a new steady state. In this case, the loss of renal function leads to a higher urea nitrogen level as well as retention of fluid. The diuretic induces loss of the excess fluid to yield a new steady state. The protein restriction reduces urea nitrogen excretion, which also leads to a new steady state. Both are adaptations. Apoptosis refers to single cell necrosis in response to injury. An irreversible injury leads to cell death, but the changes described here are not evidence for cellular necrosis. The metabolism of cells is maintained for adaptation, with response to the diuretic and to protein restriction.

1. A study of peripheral blood smears shows that neutrophil nuclei of women have a Barr body, whereas those of men do not. The Barr body is an inactivated X chromosome. Which of the following forms of RNA is most likely to play a role in Barr body formation? A. lncRNA C. mRNA B. miRNA D. siRNA E. tRNA

1. A There are forms of noncoding RNA that play a role in gene expression. Long noncoding RNA (lncRNA) segments greater than 200 nucleotides in length can bind to chromatin to restrict access of RNA polymerase to coding segments. The X chromosome transcribes XIST, a lncRNA that binds to and represses X chromosome expression. However, not all genes on the "inactive" X chromosome are switched off. The RNA transcribed from nuclear DNA that directs protein synthesis through translation is mRNA. MicroRNAs (miRNAs) are noncoding RNA sequences that inhibit the translation of mRNAs. Gene-silencing RNAs (small interfering RNAs [siRNAs]) have the same function as miRNAs, but they are produced synthetically for experimental purposes. Transfer RNA (tRNA) participates in the translation of mRNA to proteins by linking to specific amino acids.

1. An 11-year-old child falls and cuts his hand. The wound becomes infected. Bacteria extend into the extracellular matrix around capillaries. In the inflammatory response to this infection, which of the following cells removes the bacteria? A B lymphocyte B Fibroblast C Macrophage D Mast cell E T lymphocyte

1. C Macrophages in tissues derived from circulating blood monocytes are phagocytic cells that respond to a variety of stimuli, and they represent the janitorial crew of the body. The other cells listed are not phagocytes. B cells can differentiate into plasma cells secreting antibodies to neutralize infectious agents. Fibroblasts form collagen as part of a healing response. Mast cells can release a variety of inflammatory mediators. T cells are a key part of chronic inflammatory processes in cell-mediated immune responses.

10. In an experiment, neutrophils collected from peripheral blood are analyzed for a "burst" of oxygen consumption. This respiratory burst is an essential step for which of the following events in an acute inflammatory response? A Attachment to endothelial cells B Generation of microbicidal activity C Increased production in bone marrow D Opsonization of bacteria E Phagocytosis of bacteria

10. B The respiratory, or oxidative, burst of neutrophils generates reactive oxygen species (e.g., superoxide anion) that are important in destruction of engulfed bacteria. This burst can be quantitated by flow cytometric analysis. Neutrophil attachment to endothelium is aided by adhesion molecules on both the endothelium and the neutrophil surface. These molecules include selectins and integrins. Myelopoiesis does not depend on generation of superoxide. Bacteria are opsonized by complement C3b and IgG, allowing the bacteria to be more readily phagocytosed.

10. A 33-year-old woman has had increasing lethargy and decreased urine output for the past week. Laboratory studies show her serum creatinine is 4.3 mg/dL and urea nitrogen 40 mg/dL. A renal biopsy is performed, and the specimen is examined using electron microscopy. Which of the following morphologic cellular changes most likely suggests a diagnosis of acute tubular necrosis? A Chromatin clumping B Mitochondrial swelling C Nuclear fragmentation D Plasma membrane blebs E Ribosomal disaggregation

10. C Cell death occurs with loss of the cell nucleus, and tubular cells become necrotic. All other cellular morphologic changes listed represent forms of reversible cellular injury. The plasma membrane and intracellular organelles retain some function unless severe damage causes loss of mem- brane integrity.

10. In an experiment, surgical incisions are made in a study group of laboratory rats. Observations about the wounds are recorded over a 2-week period using various chemical mediators. Which of the following steps in the inflammatory-repair response is most likely affected by neutralization of transforming growth factor β (TGF-β)? A. Chemotaxis of lymphocytes B. Increase in vascular permeability C. Leukocyte extravasation D. Migration of epithelial cells E. Production of collagen

10. E TGF-β stimulates many steps in fibrogenesis, including fibroblast chemotaxis and production of collagen by fibroblasts, while inhibiting degradation of collagen. All of the other steps listed are unaffected by TGF-β.

11. A 50-year-old man has experienced an episode of chest pain for 6 hours. A representative histologic section of his left ventricular myocardium is shown in the figure. There is no hemorrhage or inflammation. Which of the following conditions most likely produced these myocardial changes? A Arterial thrombosis B Autoimmunity C Blunt chest trauma D Protein-deficient diet E Viral infection

11. A The figure shows deep eosinophilic staining, loss of myocardial fiber nuclei, and loss of cell structure consistent with an early ischemic injury, resulting in coagulative necrosis. Myocardial ischemia and infarction are typically caused by loss of coronary arterial blood flow. An immunological process may produce focal myocardial injury. Blunt trauma produces hemorrhage. Lack of protein leads to a catabolic state with gradual decrease in cell size, but it does not cause ischemic changes. Viral infection could cause focal necrosis of the myocardium, but this is usually accompanied by an inflammatory infiltrate consisting of lymphocytes and macrophages.

11. A 62-year-old man has had increasing knee pain with movement for the past 10 years. The knee joint surfaces are eroded and the joint space narrowed. There is loss of compressibility and lubrication of articular cartilaginous surfaces. Loss of which of the following extracellular matrix components has most likely occurred in this man? A. Elastin B. Fibronectin C. Hyaluronan D. Integrin E. Laminin

11. C He has osteoarthritis, or degenerative joint disease, with loss of articular hyaline cartilage. Hyaluronan (hyaluronic acid) is a large mucopolysaccharide, one form of proteoglycan, which forms a hydrated, compressible gel contributing to the shock-absorbing function of joint surfaces. Elastin is a fibrillar protein that provides recoil in tissues such as skin, arterial walls, and ligaments that need to stretch and return to their original shape. Fibronectin is a form of glycoprotein that serves an adhesive function. Integrins are glycoproteins that serve as cellular receptors for extracellular matrix components; they can link to intracellular actin so that cells can alter their shape and mobility. Laminins are a form of glycoprotein that help to anchor epithelial surfaces in basement membranes.

11. A 4-year-old girl has had numerous infections with Staphylococcus aureus since infancy. Genetic testing shows a defect leading to a lack of β2 integrin production. Which of the following abnormalities of neutrophil function is most likely responsible for these clinical symptoms? A Decreased generation of hydroxy-halide radicals (HOCl-) B Diminished phagocytosis of bacteria opsonized with IgG C Failure of migration to the site of infection D Inadequate adhesion on cytokine-activated endothelium E Reduced respiratory burst after phagocytosis

11. D During acute inflammation, in the first stage of extravasation, the neutrophils "roll over" the endothelium. At this stage, the adhesion between the neutrophils and endothelial cells is weak. Rolling is mediated by binding of selectins to sialylated oligosaccharides. The next step, firm adhesion, is mediated by binding of integrins on the leukocytes to their receptors, intercellular adhesion molecule-1 or vascular cell adhesion molecule-1 (VCAM-1), on endothelial cells. Integrins have two chains, α and β. A genetic lack of β chains prevents firm adhesion of leukocytes to endothelial cells. This process depends on adhesion molecules expressed on the neutrophils and endothelial cells. Formation of HOCl- re- quires myeloperoxidase released from neutrophil granules. Phagocytosis of opsonized organisms depends on engulfment, which requires contractile proteins in the neutrophil cytoplasm. Neutrophil migration to a site of infection depends on the presence of chemotactic factors such as complement C5a that bind to the neutrophil and activate phospholipase C to begin a series of events that culminate in the influx of calcium, which triggers contractile proteins. The respiratory burst to kill phagocytized organisms depends on NADPH oxidase, and a deficiency of this enzyme leads to chronic granulomatous disease.

12. A 38-year-old woman has experienced severe abdominal pain over the past day. On examination she is hypotensive and in shock. Laboratory studies show elevated serum lipase. From the representative gross appearance of the mesentery shown in the figure, which of the following events has most likely occurred? A Acute pancreatitis B Gangrenous cholecystitis C Hepatitis B virus infection D Small intestinal infarction E Tuberculous lymphadenitis

12. A The many focal, chalky white deposits in the mesentery, composed mainly of adipocytes, are areas of fat necrosis. The deposits result from the release of pancreatic enzymes such as lipases in a patient with acute pancreatitis. Gangrenous necrosis is mainly coagulative necrosis, but occurs over an extensive area of tissues. Viral hepatitis does not cause cell necrosis in organs other than liver, and hepatocyte necrosis from viral infections occurs mainly by means of apoptosis. Intestinal infarction is a form of coagulative necrosis. Infection with tuberculosis leads to caseous necrosis.

12. An experiment is conducted involving cellular aspects of wound healing. Components of the extracellular matrix are analyzed to determine their sites of production and their binding patterns to other tissue components. Which of the following molecules synthesized by fibroblasts can best bind to cellular integrins and extracellular collagen and attach epidermal basal cells to basement membrane? A. Dermatan sulfate B. Fibronectin C. Heparin D. Hyaluronic acid E. Procollagen

12. B Fibronectin is a key component of the extracellular matrix and has a structure that looks like a paper clip. Fibronectin can be synthesized by monocytes, fibroblasts, and endothelium. Dermatan sulfate, a glycosaminoglycan, forms a gel that provides resilience and lubrication. Heparin that is infused has an anticoagulant function. Hyaluronic acid binds water to form a gelatinous extracellular matrix. Procollagen produced by fibroblasts is formed into ropelike strands of collagen, which provide tensile strength.

12. In an experiment, peripheral blood cells are isolated and placed into a culture medium that preserves their metabolic activity. Interferon-γ is added to this culture, along with viable Escherichia coli organisms. Which of the following blood cell types in this medium is the most likely to have bactericidal activity against E. coli? A Basophil B B lymphocyte C CD4+ lymphocyte D CD8+ lymphocyte E Monocyte F Natural killer cell G Neutrophil

12. E Monocytes transforming to macrophages contain cytokine-inducible nitric oxide synthase (iNOS), which generates nitric oxide. Nitric oxide, by itself and on interaction with other reactive oxygen species, has antimicrobial activity. CD4 or CD8 lymphocytes can be the source for interferon-γ (IFN-γ), which stimulates macrophage production of NOS. Endothelial cells contain a form of NOS (eNOS) that acts to promote vasodilation. B lymphocytes produce immunoglobulins that can opsonize bacteria. Basophils release histamine and arachidonic acid metabolites, which participate in the acute inflammatory process. Natural killer cells have Fc receptors and can lyse IgG-coated target cells; they also generate IFN-γ. Neutrophils can phagocytize microbes, but they use NAPDH oxidase and enzymes other than NOS to kill the microbes.

13. In an experiment, T lymphocytes from peripheral blood are placed in a medium that preserves their function. The lymphocytes are activated by contact with antigen and incubated for 4 hours. The supernatant fluid is collected and is found to contain a substance that is a major stimulator of monocytes and macrophages. Which of the following substances released into this fluid medium is most likely to stimulate macrophages? A Histamine B Interferon-γ C Leukotriene B4 D Nitric oxide E Phospholipase C F Tumor necrosis factor (TNF)

13. B Interferon-γ secreted from lymphocytes stimulates monocytes and macrophages, which secrete their own cytokines that further activate lymphocytes. Interferon-γ also is important in transforming macrophages into epithelioid cells in a granulomatous inflammatory response. Histamine released from mast cells is a potent vasodilator, increasing vascular permeability. Leukotriene B4, generated in the lipoxygenase pathway of arachidonic acid metabolism, is a potent neutrophil chemotactic factor. Nitric oxide generated by macrophages aids in destruction of microorganisms; nitric oxide released from endothelium mediates vasodilation and inhibits platelet activation. Binding of agonists such as epinephrine, collagen, or thrombin to platelet surface receptors activates phospholipase C, which catalyzes the release of arachidonic acid from two of the major membrane phospholipids, phosphatidylinositol and phosphatidylcholine. Tumor necrosis factor (TNF), produced by activated macrophages, mediates many systemic effects, including fever, metabolic wasting, and hypotension.

13. An experiment analyzes factors involved in the cell cycle during growth factor-induced cellular regeneration in a tissue culture. Cyclin B synthesis is induced; the cyclin B binds and activates cyclin-dependent kinase 1 (CDK1). The active kinase produced by this process is most likely to control progression in which of the following phases of the cell cycle? A. G0 to G1 B. G1 to S C. S to G2 D. G2 to M E. M to G1

13. D CDK1 controls an extremely important transition point, the G2 to M transition, during the cell cycle, which can be regulated by CDK inhibitors. The other checkpoints listed are regulated by a distinct set of proteins.

13. A 68-year-old woman suddenly lost consciousness and on awakening 1 hour later, she could not speak or move her right arm. Two months later, a head CT scan showed a large cystic area in the left parietal lobe. Which of the following pathologic processes has most likely occurred in her brain? A Apoptosis B Coagulative necrosis C Fat necrosis D Karyolysis E Liquefactive necrosis

13. E The high lipid content of central nervous system tissues results in liquefactive necrosis as a consequence of ischemic injury, as in this case of stroke. Apoptosis affects single cells and typically is not grossly visible. Coagulative necrosis is the typical result of ischemia in most solid organs. Fat necrosis is seen in breast and pancreatic tissues. Karyolysis refers to fading away of cell nuclei in dead cells.

14. In an experiment, the role of low-density lipoprotein (LDL) receptors in uptake of lipids in the liver is studied. A mouse model is created in which the LDL receptor gene is not expressed in the liver. For creating such a knockout mouse, which of the following cells would be most useful? A. Adult bone marrow mesenchymal progenitor cells B. Embryonic stem cells in culture C. Hematopoietic stem cells D. Hepatic oval cells E. Regenerating hepatocytes

14. B Embryonic stem (ES) cells are multipotent and can give rise to all cells, including hepatocytes. Gene targeting to produce knockout mice is done in cultures of ES cells, which are then injected into mouse blastocysts and implanted into the uterus of a surrogate mother. Mesenchymal stem cells also are multipotential, but they are not useful for gene targeting. Hematopoietic stem cells can give rise to all hematopoietic cells, but not other types of cells. Hepatocytes and oval cells within the liver can give rise only to liver cells.

14. A screening chest radiograph of an asymptomatic 37-year-old man shows a 3-cm nodule in the middle lobe of his right lung. The nodule is excised with a pulmonary wedge resection, and sectioning shows a sharply circumscribed mass with a soft, white center. The microscopic appearance is shown in the figure. The serum interferon gamma release assay is positive. Which of the following pathologic processes has most likely occurred in this nodule? A Apoptosis B Caseous necrosis C Coagulative necrosis D Fat necrosis E Fatty change F Gangrenous necrosis G Liquefactive necrosis

14. B The grossly cheeselike appearance gives this form of necrosis its name—caseous necrosis. The figure shows amorphous pink acellular material at the upper right surrounded by epithelioid macrophages, and a Langhans giant cell is visible at the upper left. In the lung, tuberculosis and fungal infections are most likely to produce this pattern of tissue injury. Apoptosis involves individual cells, without grossly apparent extensive or localized areas of tissue necrosis. Coagulative necrosis is more typical of ischemic tissue injury. Fat necrosis most often occurs in the breast and pancreas. Fatty change is most often a feature of hepatocyte injury, and the cell integrity is maintained. Gangrene characterizes extensive necrosis of multiple cell types in a body region or organ. Liquefactive necrosis is seen in neutrophilic abscesses or ischemic cerebral injury.

14. A woman who is allergic to cats visits a neighbor who has several cats. During the visit, she inhales cat dander, and within minutes, she develops nasal congestion with abundant nasal secretions. Which of the following substances is most likely to produce these findings? A Bradykinin B Complement C5a C Histamine D Interleukin-1 (IL-1) E Phospholipase C F Tumor necrosis factor (TNF)

14. C Histamine is found in abundance in mast cells, which are normally present in connective tissues next to blood vessels beneath mucosal surfaces in airways. Binding of an antigen (allergen) to IgE antibodies that have previously attached to the mast cells by the Fc receptor triggers mast cell degranulation, with release of histamine. This response causes increased vascular permeability and mucous secretions. Bradykinin, generated from the kinin system on surface contact of Hageman factor with collagen and basement membrane from vascular injury, promotes vascular permeability, smooth muscle contraction, and pain. Complement C5a is a potent chemotactic factor for neutrophils. Interleukin-1 (IL-1) and tumor necrosis factor (TNF), both produced by activated macrophages, mediate many systemic effects, including fe- ver, metabolic wasting, and hypotension. Phospholipase C, which catalyzes the release of arachidonic acid, is generated from platelet activation.

15. An experimental drug administered to a tissue preparation is found to inhibit cellular oxidative phosphorylation when given in high doses, and ATP production drops to 5% of normal. Cell membrane function is diminished. Which of the following substances is most likely to be present at increased concentration in culture fluid bathing the tissue? A Calcium B Glucose C Ketones D Potassium E Sodium

15. D Reduction in oxidative phosphorylation leads to reduction in synthesis of ATP and diminished activity of the plasma membrane sodium pump, which maintains high intracellular potassium concentration. Loss of ATP leads to efflux of intracellular potassium, while net influx of sodium and water promote cell swelling. A marked rise in plasma potassium can indicate significant cell damage or death (such as skeletal muscle crush injury or hemolysis). When cells are not consuming glucose via oxidative metabolism, the glucose is metabolized via other pathways, and glucose is maintained within normal ranges. Though cell membranes are composed of lipid, dysfunction or disruption of those membranes does not significantly alter plasma lipid concentrations.

15. Dermal fibroblasts are harvested from the skin biopsy specimen of an adult man. These fibroblasts are transduced with genes encoding for transcription factors including SOX2 and MYC. Under appropriate culture conditions these cells are then able to generate endodermal, mesodermal, and ectoder- mal cells. Into which of the following kinds of stem cell have these fibroblasts been transformed? A. Embryonic B. Lineage-committed C. Mesenchymal D. Pleuripotent

15. D These transformed cells are designated iPS cells be- cause they have been induced to become pleuripotent. This transformation process gets around the problem of using em- bryonic stem (ES) cells derived from manipulation of human embryos, which raises ethical and religious concerns. Embry- onic stem cells are totipotent, but they become pleuripotent cells that can further divide into many different cell lines, yet maintain themselves in a replicating pool. Thus pleuripotent cells are the next best thing compared to embryonic cells for deriving human cells that could replace damaged or diseased tissues. Further differentiation of pleuripotent cells gives rise to cells with more restricted developmental capacity, such as mes- enchymal stem cells that can give rise to tissues such as muscle and cartilage but not to endodermal or ectodermal cells.

16. A 19-year-old woman develops a sore throat and fever during the past day. Physical examination shows pharyngeal erythema and swelling. Laboratory findings include leukocytosis. She is given naproxen. Which of the following features of the acute inflammatory response is most affected by this drug? A Chemotaxis B Emigration C Leukocytosis D Phagocytosis E Vasodilation

16. E Naproxen, a nonsteroidal anti-inflammatory drug, targets the cyclooxygenase pathway of arachidonic acid metabolism and leads to reduced prostaglandin generation. Prostaglandins promote vasodilation at sites of inflammation. Chemotaxis is a function of various chemokines, and complement C3b may promote phagocytosis, but neither is affected by aspirin. Leukocyte emigration is aided by various adhesion molecules. Leukocyte release from bone marrow can be driven by the cytokines interleukin-1 (IL-1) and tumor necrosis factor (TNF).

15. In a 6-month randomized trial of a pharmacologic agent, one group of patients receives a cyclooxygenase-2 (COX-2) inhibitor, and a control group does not. Both groups of adult males had mild congestive heart failure and bilateral symmetric arthritis of small joints. Laboratory measurements during the trial show no significant differences between the groups in WBC count, platelet count, hemoglobin, and creatinine. The group receiving the drug reports subjective findings different from those of the control group. Which of the following findings was most likely reported by the group receiving the drug? A Increased ankle swelling B Increased susceptibility to bruising C Increased bouts of asthma D Reduced severity of urticaria E Numerous febrile episodes F Reduced arthritis pain

15. F The COX-2 enzyme is inducible with acute inflammatory reactions, particularly in neutrophils, in synovium, and in the central nervous system. The cyclooxygenase pathway of arachidonic acid metabolism generates prostaglandins, which mediate pain, fever, and vasodilation. Ankle swelling is most likely to result from peripheral edema secondary to congestive heart failure. Increased susceptibility to bruising results from prolonged glucocorticoid administration, which also causes leukopenia. Asthma results from bronchoconstriction mediated by leukotrienes that are generated by the lipoxygenase pathway of arachidonic acid metabolism. Inhibition of histamine released from mast cells helps reduce urticaria. Fever can be mediated by prostaglandin release, not inhibition.

16. A 47-year-old woman has poorly controlled diabetes mellitus and develops coronary artery disease. She now has decreasing cardiac output with blood pressure of 80/40 mm Hg and ejection fraction of 18%. An increase in which of the following substances in her blood is most indicative of reversible cell injury from decreased systemic arterial perfusion of multiple organs and tissues? A Carbon dioxide B Creatinine C Glucose D Lactic acid E Troponin I

16. D Decreased tissue perfusion from hypotensive shock leads to hypoxemia and depletion of ATP when cell metabolism shifts from aerobic to anaerobic glycolysis. This shift causes depletion of glycogen stores and increased production and accumulation of lactic acid, reducing intracellular pH. Creatinine would increase with reduced renal function from decreased renal perfusion, but this would not explain the changes in other tissues. An increased glucose level would be indicative of poorly controlled diabetes mellitus, not decreased perfusion. Carbon dioxide is likely to be cleared via normal lungs, which are still sufficiently perfused by a failing heart. An increase in troponin I suggests irreversible myocardial injury.

17. A tissue preparation is experimentally subjected to a hypoxic environment. The cells in this tissue begin to swell, and chromatin begins to clump in cell nuclei. ATPases are activated, and ATP production decreases. Which of the following ions accumulating in mitochondria and the cytosol contributes most to these findings and to eventual cell death? A Ca2+ B Cl− C HCO3− D K+ E Na+ F PO43−

17. A Irreversible cellular injury is likely to occur when cytoplasmic calcium increases. Calcium can enter cells and also accumulate in mitochondria and endoplasmic reticulum. The excess calcium activates ATPases, phospholipases, proteases, and endonucleases, which injure cell components. Mitochondrial permeability is increased to release cytochrome c, which activates caspases leading to apoptosis. Of the other ions listed, sodium enters the cell, while potassium diffuses out when the sodium pump fails as ATP levels fall; but this is potentially reversible.

17. A 35-year-old woman takes acetylsalicylic acid (aspirin) for arthritis. Although her joint pain is reduced with this therapy, the inflammatory process continues. The aspirin therapy alleviates her pain mainly through reduction in the synthesis of which of the following mediators? A Complement C1q B Histamine C Leukotriene E4 D Nitric oxide E Prostaglandins

17. E Prostaglandins are produced through the cyclooxygenase pathway of arachidonic acid metabolism. Aspirin and other nonsteroidal anti-inflammatory drugs block the synthesis of prostaglandins, which can produce pain. Complement C1q is generated in the initial stage of complement activation, which can eventually result in cell lysis. Histamine is mainly a vasodilator. Leukotrienes are generated by the lipoxygenase pathway, which is not blocked by aspirin. Nitric oxide released from endothelium is a vasodilator.

18. In an experiment, a large amount of a drug is administered to experimental organisms and is converted by cytochrome P-450 to a toxic metabolite. Accumulation of this metabolite leads to increased intracellular lipid peroxidation. Depletion of which of the following intracellular substances within the cytosol exacerbates this form of cellular injury by this mechanism? A ADP B Glutathione C NADPH oxidase D Nitric oxide synthase E mRNA F Sodium

18. B The drug acetaminophen can be converted to toxic metabolites in this manner. Glutathione in the cytosol helps to reduce cellular injury from many toxic metabolites and free radicals. ADP is converted to ATP by oxidative and glycolytic cellular pathways to provide energy that drives cellular functions, and a reduction in ATP leaves the cell vulnerable to injury. NADPH oxidase generates superoxide, which is used by neutrophils in killing bacteria. Nitric oxide synthase in macrophages produces nitric oxide, which aids in destroying organisms undergoing phagocytosis. Protein synthesis in cells depends on mRNA for longer survival and recovery from damage caused by free radicals. Failure of the sodium pump leads to increased cytosolic sodium and cell swelling with injury.

18. A 77-year-old woman experiences a sudden loss of consciousness, with loss of movement on the right side of the body. Cerebral angiography shows an occlusion of the left middle cerebral artery. Elaboration of which of the following mediators will be most beneficial in preventing further ischemic injury to her cerebral cortex? A Bradykinin B Leukotriene E4 C Nitric oxide D Platelet-activating factor E Thromboxane A2

18. C Endothelial cells can release nitric oxide to promote vasodilation in areas of ischemic injury. Bradykinin mainly increases vascular permeability and produces pain. Leuko- triene E4, platelet-activating factor, and thromboxane A2 have vasoconstrictive properties.

19. In an experiment, bacteria are inoculated into aliquots of normal human blood that have been treated with an anticoagulant. It is observed that the bacteria are either phagocytized by neutrophils or undergo lysis. Which of the following blood plasma components is most likely to facilitate these effects? A Complement B Fibrin C Kallikrein D Plasmin E Thrombin

19. A Activation of complement may occur via microbial cell wall components such as polysaccharides (alternative pathway) or mannose (lectin pathway), or antibody attached to surface antigens (classic pathway). A variety of complement components are generated, including complement C5a, a neutrophil chemoattractant; complement C3b, an opsonin; and complement C5-9, the membrane attack complex. The remaining options are more closely associated with coagulation. Fibrin is generated by the coagulation system, but not with anticoagulation. Kallikrein may aid in generation of bradykinin and plasmin, but participates just in complement C5a generation. Plasmin is generated from plasminogen and helps lyse clots. Thrombin is generated by the coagulation cascade.

19. In an experiment, metabolically active cells are subject- ed to radiant energy in the form of x-rays. This results in cell injury caused by hydrolysis of water. Which of the following intracellular enzymes helps to protect the cells from this type of injury? A Endonuclease B Glutathione peroxidase C Lactate dehydrogenase D Phospholipase E Protease

19. B The body has intracellular mechanisms that prevent damage from free radicals generated by exposure to x-rays. Glutathione peroxidase reduces such injury by catalyzing the breakdown of hydrogen peroxide. Endonucleases damage DNA in nuclear chromatin. Lactate dehydrogenase is present in a variety of cells, and its elevation in the serum is an indicator of cell injury and death. Phospholipases decrease cellular phospholipids and promote cell membrane injury. Proteases can damage cell membranes and cytoskeletal proteins.

2. In an experiment, a nuclear chromosomal gene is found to be actively transcribing messenger RNA (mRNA) that is transported into the cell cytoplasm. However, there is no observed protein product from translation of this mRNA. How is the silencing of this active gene's mRNA most likely to occur? A. Absence of tRNA B. Binding to miRNA C. Methylation of DNA D. Mutation of mRNA E. Upregulation of mtDNA

2. B MicroRNAs (miRNA) are encoded by about 5% of the human genome. miRNAs do not encode for proteins, but bind to and inactivate or cleave to mRNA, preventing translation of proteins by mRNA, effectively silencing gene expression without affecting the gene directly. There is abundant tRNA present in the cytoplasm that is not a rate-limiting step to translation. DNA methylation, particularly at CG dinucleotides, is a way of suppressing gene expression directly, as is seen with genomic imprinting. Mutations that occur in genes in DNA may result in reduced mRNA production or abnormal protein production, but mRNA itself is not mutated. Mitochondrial DNA (mtDNA) encodes for proteins mainly involved in oxidative phosphorylation metabolic pathways.

2. A 53-year-old woman has had a high fever and cough productive of yellowish sputum for the past 2 days. Her vital signs include temperature of 37.8° C, pulse 103/min, respirations 25/min, and blood pressure 100/60 mm Hg. On auscultation of the chest, crackles are audible in both lung bases. A chest radiograph shows bilateral patchy pulmonary infiltrates. The microscopic appearance of her lung is shown in the figure. Which of the following inflammatory cell types is most likely to be seen in greatly increased numbers in her sputum specimen? A Langhans giant cells B Macrophages C Mast cells D Neutrophils E T lymphocytes

2. D These signs and symptoms suggest acute bacterial pneumonia. Such infections induce an acute inflammation dominated by neutrophils that fill alveoli, as shown in the figure, and are coughed up, which gives the sputum its yellowish, purulent appearance. Langhans giant cells are seen with granulomatous inflammatory responses. Macrophages become more numerous after initiation of acute events, cleaning up tissue and bacterial debris through phagocytosis. Mast cells are better known as participants in allergic and anaphylactic responses. Lymphocytes are a feature of chronic inflammation.

2. A 53-year-old woman with no prior illnesses has a routine checkup by her physician. On examination she has a blood pressure of 150/95 mm Hg. If her hypertension remains untreated for years, which of the following cellular alterations would most likely be seen in her myocardium? A Apoptosis B Dysplasia C Fatty change D Hemosiderosis E Hyperplasia F Hypertrophy G Metaplasia

2. F The pressure load on the left ventricle results in an increase in myofilaments in the existing myofibers, so they enlarge. The result of continued stress from hypertension is eventual heart failure with decreased contractility. Apoptosis would lead to loss of cells and diminished size. Dysplasia is not a diagnosis made for the heart. Hemosiderin deposition in the heart is a pathologic process resulting from increased iron stores in the body. Though hyperplasia from proliferation of myofibroblasts is possible, this does not contribute significantly to cardiac size. Metaplasia of muscle does not occur, although loss of muscle occurs with aging and ischemia as myofibers are replaced by fibrous tissue.

22. A 24-year-old, sexually active woman has experienced lower abdominal pain for the past day. Her temperature is 37.9° C, and on palpation, the left lower abdomen is markedly tender. Laboratory findings include a total WBC count of 29,000/mm3 with 75% segmented neutrophils, 6% bands, 14% lymphocytes, and 5% monocytes. Laparotomy reveals a dis- tended, fluid-filled, reddened left fallopian tube that is about to rupture. A left salpingectomy is performed. Which of the following is most likely to be seen on microscopic examination of the excised fallopian tube? A. Fibroblastic proliferation B. Langhans giant cells C. Liquefactive necrosis D. Mononuclear infiltrates E. Squamous metaplasia

22. C This patient is experiencing an acute inflammatory response, with edema, erythema, and pain of short duration. Neutrophils form an exudate and release various proteases, which can produce liquefactive necrosis, starting at the mucosa and extending through the wall of the tube. This mechanism results in perforation. Fibroblasts are more likely participants in chronic inflammatory responses and in healing responses, generally appearing more than 1 week after the initial event. Langhans giant cells are a feature of granulomatous inflammation. Mononuclear infiltrates are more typical of chronic inflammation of the fallopian tube, in which rupture is less likely. Epithelial metaplasia is most likely to occur in the setting of chronic irritation with inflammation.

22. A 54-year-old man experienced severe substernal chest pain for 3 hours. An ECG showed changes consistent with an acute myocardial infarction. After thrombolytic therapy with tissue plasminogen activator (t-PA), his serum creatine kinase (CK) level increased. Which of the following tissue events most likely occurred in the myocardium after t-PA therapy? A Cellular regeneration B Drug toxicity C Increased synthesis of CK D Myofiber atrophy E Reperfusion injury

22. E If existing cell damage is not great after myocardial infarction, the restoration of blood flow can help prevent further cellular damage. However, the reperfusion of damaged cells results in generation of oxygen-derived free radicals, causing a reperfusion injury. The elevation in the CK level is indicative of myocardial cell necrosis, because this intracellular enzyme does not leak in large quantities from intact myocardial cells. Myocardial fibers do not regenerate to a significant degree, and atrophic fibers would have less CK to release. t-PA does not produce a toxic chemical injury; it induces thrombolysis to restore blood flow in occluded coronary arteries.

23. On day 28 of her menstrual cycle, a 23-year-old woman experiences onset of menstrual bleeding that lasts for 6 days. She has had regular cycles since menarche. Which of the following processes most likely occurs in her endometrial cells to initiate the onset of menstrual bleeding? A Apoptosis B Atrophy C Caseous necrosis D Heterophagocytosis E Liquefactive necrosis

23. A The onset of menstruation is orderly, programmed cell death (apoptosis) through hormonal stimuli, an ex- ample of the intrinsic (mitochondrial) apoptotic pathway. As hormone levels drop, the endometrium breaks down, sloughs off, and then regenerates. With cellular atrophy, there is often no visible necrosis, but the tissues shrink, something that occurs in the endometrium after menopause. Caseous necrosis is typical of granulomatous inflammation, resulting most commonly from mycobacterial infection. Heterophagocytosis is typified by the clearing of an area of necrosis through macrophage ingestion of the necrotic cells. Liquefactive necrosis can occur in any tissue after acute bacterial infection or in the brain after ischemia.

23. A 68-year-old man has had worsening shortness of breath for the past week. On physical examination, his temperature is 38.3° C. On percussion, there is dullness over the left lung fields. Thoracentesis performed on the left pleural cavity yields 800 mL of cloudy yellow fluid that has a WBC count of 2500/mm3 with 98% neutrophils and 2% lymphocytes. A Gram stain of the fluid shows gram-positive cocci in clusters. Which of the following terms best describes the pro- cess occurring in his left pleural cavity? A Abscess B Chronic inflammation C Edema D Fibrinous inflammation E Purulent exudate F Serous effusion

23. E Bacterial infections often evoke an acute inflammatory response dominated by neutrophils. The extravasated neutrophils attempt to phagocytose and kill the bacteria. In the pro- cess, some neutrophils die, and the release of their lysosomal enzymes can cause liquefactive necrosis of the tissue. This liquefied tissue debris and both live and dead neutrophils comprise pus, or purulent exudate. Such an exudate is typical of bacterial infections that involve body cavities. Another term for purulent exudate in the pleural space is empyema. An abscess is a localized collection of neutrophils within tissues. Chronic inflammation occurs when there is a preponderance of mononuclear cells, such as lymphocytes, macrophages, and plasma cells, in a process that has gone on for more than a few days—more likely weeks or months—or that accompanies repeated bouts of acute inflammation. Edema refers to increased cellular and interstitial fluid collection within tissues, leading to tissue swelling. In fibrinous inflammation, exudation of blood proteins (including fibrinogen, which polymerizes to fibrin) gives a grossly shaggy appearance to surfaces overly- ing the inflammation. A serous effusion is a watery-appearing transudate that resembles an ultrafiltrate of blood plasma, with a low cell and protein content.

24. An experiment introduces a knockout gene mutation into a cell line. The frequency of shrunken cells with chromatin clumping, karyorrhexis, and cytoplasmic blebbing is increased compared with a cell line without the mutation. Overall survival of the mutant cell line is reduced. Which of the following genes is most likely to be affected by this mutation? A BAX B BCL2 C C-MYC D FAS E p53

24. B These histologic findings are typical of apoptosis. The BCL2 gene product inhibits cellular apoptosis by bind- ing to Apaf-1. Hence, the knockout removes this inhibition The BAX gene product promotes apoptosis, and a knockout would protect against apoptosis. The C-MYC gene is involved with oncogenesis. The FAS gene encodes for a cellular receptor for Fas ligand that signals apoptosis. Activity of the p53 (TP53) gene normally stimulates apoptosis, but mutation favors cell survival.

24. An 87-year-old woman has had a cough productive of yellowish sputum for the past 2 days. On examination her temperature is 37° C. A chest radiograph shows bilateral patchy infiltrates. Her peripheral blood shows leukocytosis. A week later she is afebrile. Which of the following is the most likely outcome of her pulmonary disease? A Chronic inflammation B Fibrous scarring C Neoplasia D Resolution E Ulceration

24. D If inflammation is limited and brief, and the involved tissue can regenerate, then resolution is the likely outcome, without significant loss of function. In older per- sons this may take longer, but can still occur. Multiple bouts of acute inflammation, or ongoing inflammation, can become chronic, and there tends to be loss of some tissue function. If significant tissue destruction occurs, there is likely to be formation of a fibrous scar in the region of the tissue loss. Acute inflammation is not a preneoplastic event. Ulceration refers to loss of an epithelial surface with acute inflammation; if the epithelium regenerates, then there is resolution.

25. A 22-year-old woman with leukemia undergoes bone marrow transplantation and receives a partially mismatched donor marrow. One month later, she has a scaling skin rash. A skin biopsy is obtained, and on microscopic examination, it has the cellular change shown in the figure. This change most likely results from which of the following biochemical reactions? A Activation of caspases B Elaboration of lipases C Increase in glycolysis D Peroxidation of lipids E Reduction of ATP synthesis

25. A There is an apoptotic cell (arrow) that is shrunken and has been converted into a dense eosinophilic mass. There is a surrounding inflammatory reaction with cytotoxic lymphocytes. This pattern is typical of apoptosis. Caspase activation is a universal feature of apoptosis, regardless of the initiating cause. Apoptosis induced in recipient cells from donor lymphocytes occurs with graft-versus-host disease. Lipases are activated in enzymatic fat necrosis. Reduced ATP synthesis and increased glycolysis occur when a cell is subjected to anoxia, but these changes are reversible. Lipid peroxidation occurs when the cell is injured by free radicals.

25. A 53-year-old woman has experienced abdominal pain for 2 weeks. She is afebrile. There is mild upper abdominal tenderness on palpation, and bowel sounds are present. An upper gastrointestinal endoscopy is performed. The figure shows microscopic examination of a biopsy specimen of a duodenal lesion. Which of the following pathologic processes is most likely present? A Abscess B Caseating granuloma C Chronic inflammation D Purulent exudate E Serous effusion F Ulceration

25. F Inflammation involving an epithelial surface may cause such extensive necrosis that the surface becomes eroded, forming an ulcer. If the inflammation continues, the ulcer can continue to penetrate downward into submucosa and muscularis. Alternatively, the ulcer may heal, or it may remain chronically inflamed. An abscess is a localized collection of neutrophils in tissues. A caseating granuloma is granulomatous inflammation with central necrosis; the necrosis has elements of both liquefaction and coagulative necrosis. Chronic inflammation occurs when there is a preponderance of mononuclear cells, such as lymphocytes, macrophages, and plasma cells, in a process that has gone on for more than a few days—more likely weeks or months—or that accompanies repeated bouts of acute inflammation. Pus, or a purulent exudate, appears semiliquid and yellowish because of the large numbers of granulocytes present. A serous effusion is a watery-appearing transudate that resembles an ultrafiltrate of blood plasma, with a low cell and protein content.

26. A 92-year-old woman is diagnosed with Staphylococcus aureus pneumonia and receives a course of antibiotic therapy. Two weeks later, she no longer has a productive cough, but she still has a temperature of 38.1° C. A chest radiograph shows the findings in the figure. Which of the following terms best describes the outcome of the patient's pneumonia? A Abscess formation B Complete resolution C Fibrous scarring D Chronic inflammation E Tissue regeneration

26. A The rounded density in the right lower lobe of the lung has liquefied contents that form a central air-fluid level. There are surrounding infiltrates. The formation of a fluid- filled cavity after infection with Staphylococcus aureus suggests that liquefactive necrosis has occurred. The cavity is filled with tissue debris and viable and dead neutrophils (pus). Localized, pus-filled cavities are called abscesses. Some bacterial organisms, such as S. aureus, are more likely to be pyogenic, or pus-forming. With complete resolution, the structure of the lung remains almost unaltered. Scarring or fibrosis may follow acute inflammation as the damaged tissue is replaced by fibrous connective tissue. Most bacterial pneumonias re- solve, and progression to continued chronic inflammation is uncommon. Lung tissue, in contrast to liver, is incapable of regeneration, except for epithelium and endothelium.

26. A 47-year-old man has a lung carcinoma with metas- tases. He receives chemotherapy. A month later, histologic examination of a metastatic lesion shows many foci in which individual tumor cells appear shrunken and deeply eosinophilic. Their nuclei exhibit condensed aggregates of chromatin under the nuclear membrane. The pathologic process affecting these shrunken tumor cells is most likely triggered by release of which of the following substances into the cytosol? A BCL2 B Catalase C Cytochrome c D Lipofuscin E Phospholipase

26. C This histologic picture is typical of apoptosis produced by chemotherapeutic agents. The release of cytochrome c from the mitochondria is a key step in many forms of apoptosis, and it leads to the activation of caspases. BCL2 is an antiapoptotic protein that prevents cytochrome c release and prevents caspase activation. Catalase is a scavenger of hydrogen peroxide. Lipofuscin is a pigmented residue representing

27. In a study of viral hepatitis infection, it is observed that cytotoxic T lymphocytes (CTLs) induce death in virally infected hepatocytes. The CTLs release perforin to allow entry of their granules. Which of the following substances is found in those granules that directly activates programmed cell death? A BCL2 B Endonuclease C Granzyme B D Nitric oxide E p53

27. C Granzyme B is a serine protease found in CTLs that can directly trigger apoptosis. CTLs express Fas ligand on their surfaces, and when contacting Fas receptors on the target cell, the ligand can induce apoptosis by the extrinsic (death receptor-initiated) pathway. BCL2 favors cell survival. Nitric oxide helps destroy phagocytized microbes. Endonucleases are generated following caspase activation and lead to nuclear fragmentation. When p53 is activated by intrinsic DNA damage during cell proliferation, apoptosis is triggered. Mutations in p53 may allow accumulation of genetic damage, a process that promotes unregulated cell growth (neoplasia).

27. A 29-year-old woman with a congenital ventricular septal defect has had a persistent temperature of 38.6° C and headache for the past 3 weeks. A head CT scan shows an enhancing 3-cm, ring like lesion in the right parietal lobe of her brain. Which of the following actions by inflammatory cells has most likely produced this CT finding? A Elaboration of nitric oxide by macrophages B Formation of immunoglobulin by B lymphocytes C Generation of prostaglandin by endothelium D Production of interferon-γ by T lymphocytes E Release of lysosomal enzymes from neutrophils

27. E This patient has an infective endocarditis with septic embolization, producing a cerebral abscess. The tissue destruction that accompanies abscess formation as part of acute inflammatory processes occurs from lysosomal enzymatic destruction, aided by release of reactive oxygen species. Nitric oxide generated by macrophages aids in destruction of infectious agents. Immunoglobulin formed by B cells neutralizes and opsonizes infectious agents. Prostaglandins produced by endothelium promote vasodilation. Interferon-γ released from lymphocytes plays a major role in chronic and granulomatous inflammatory responses.

28. A 37-year-old man has had midepigastric pain for the past 3 months. An upper gastrointestinal endoscopy shows a 2-cm, sharply demarcated, shallow ulceration of the gastric antrum. Microscopic examination of a biopsy from the ulcer base shows angiogenesis, fibrosis, and mononuclear cell in- filtrates with lymphocytes, macrophages, and plasma cells. Which of the following terms best describes this pathologic process? A Acute inflammation B Chronic inflammation C Fibrinous inflammation D Granulomatous inflammation E Serous inflammation

28. B One outcome of acute inflammation with ulceration is chronic inflammation. This is particularly true when the inflammatory process continues for weeks to months. Chronic inflammation is characterized by tissue destruction, mononuclear cell infiltration, and repair. In acute inflammation, the healing process of fibrosis and angio- genesis has not begun. In fibrinous inflammation, typically involving a mesothelial surface, there is an outpouring of protein-rich fluid that results in precipitation of fibrin. Granulomatous inflammation is a form of chronic inflammation in which epithelioid macrophages form aggregates. Serous inflammation is an inflammatory process involving a mesothelial surface (e.g., lining of the pericardial cavity), with an outpouring of fluid having little protein or cellular content.

28. An experimental study of steatohepatitis in metabolic syndrome reveals that hepatocyte cell membrane injury with necrosis occurs in response to increased amounts of tumor necrosis factor (TNF). When a pharmacologic agent inhibit- ing caspases is administered, cell necrosis still occurs. Which of the following substances forms a supramolecular complex that increases the generation of reactive oxygen species? A Catalase B Cytochrome c C Interleukin 1-beta converting enzyme D Receptor-interacting protein E Ubiquitin ligase

28. D Necroptosis occurs when the mechanism of apoptosis yields morphologic necrosis following cell membrane rupture, independent of caspase release. The RIP1-RIP3 complex is called a necrosome. Catalases help destroy hydrogen peroxide to prevent free radical damage. Cytochrome c participates in apoptosis and an inflammasome in necroptosis. Ubiquitin ligase is part of misfolded protein processing in proteasomes.

29. A 71-year-old man diagnosed with pancreatic cancer is noted to have decreasing body mass index. His normal con- nective tissues undergo atrophy by sequestering organelles and cytosol in a vacuole, which then fuses with a lysosome. However, the cancer continues to increase in size. Which of the following processes is most likely occurring in the normal cells but is inhibited in the cancer cells of this man? A Aging B Apoptosis C Autophagy D Hyaline change E Karyorrhexis

29. C Autophagy is a form of cellular downsizing in response to stress, as the cell consumes itself, by upregulating Atgs genes. Lipofuscin granules are residual bodies left over from this process. Cell death may eventually be triggered by autophagy, but by a different mechanism than apoptosis, a form of single cell necrosis in which cell frag- mentation occurs. Cancer cells acquire the ability to prevent autophagy, perhaps by downregulating PTEN gene expression, and maintain a survival advantage even as the patient is dying. There is slow autophagy with aging, but autophagy is accelerated with stressors such as malnutrition and chronic disease. Hyaline is a generic term for intracellular or extracellular protein accumulations appearing pink and homogeneous with H&E staining. Karyorrhexis is nuclear fragmentation in a necrotic cell.

29. A 65-year-old man develops worsening congestive heart failure 2 weeks after an acute myocardial infarction. An echocardiogram shows a markedly decreased ejection fraction. Now, capillaries, fibroblasts, collagen, and inflammatory cells have largely replaced the infarcted myocardium. Which of the following inflammatory cell types in this lesion plays the most important role in the healing process? A Eosinophils B Epithelioid cells C Macrophages D Neutrophils E Plasma cells

29. C Macrophages, present in such lesions, play a prominent role in the healing process. Activated macrophages can secrete various cytokines that promote angiogenesis and fibrosis, including platelet-derived growth factor, fibroblast growth factor, interleukin-1 (IL-1), and tumor necrosis factor (TNF). Eosinophils are most prominent in allergic inflammations and in parasitic infections. Epithelioid cells, which are aggregations of activated macrophages, are typically seen with granulomatous inflammation, and the healing of acute inflammatory processes does not involve granulomatous inflammation. Neutrophils are most numerous within the initial 48 hours after infarction, but are not numerous after the first week. Plasma cells can secrete immunoglobulins and are not instrumental to healing of an area of tissue injury.

3. A proponent of Chilean Malbec, Syrah, and Merlot wines (all reds) touts their contribution to longevity, but this wine aficionado also controls his dietary caloric content so that his body mass index is <22. This lifestyle promotes increased insulin sensitivity and glucose utilization. He fully expects to live longer because he has read that caloric restriction prolongs life. In this man, which of the following intracellular substances will most likely mediate the effect of calorie restriction upon increased longevity? A. Caspase B. Glutathione C. Sirtuins D. Telomerase E. Ubiquitin

3. C The one sure way to increase life span is calorie restriction. But why do without the things we like, only to do without them longer? Dietary excesses lead to increased mor- bidity with reduced quality of life, as well as mortality, from chronic diseases such as diabetes mellitus. The activity of sirtuins on histone acetylation and deacetylation may promote transcription of genes encoding for proteins that increase metabolic activity and inhibit effects of free radicals. Red wines have been shown to increase sirtuins, but don't drink too much! Moderation is the key. Glutathione promotes free radical breakdown, although chronic excessive alcohol consumption depletes hepatocyte glutathione. Caspases trigger apoptosis and cell death. Telomerases aid in promoting continued cell division, but cannot be altered by lifestyle, and turning them on is one feature of neoplasia. Ubiquitin is a peptide that is part of the ubiquitin-proteasome pathway of protein degradation seen with nutrient deficiencies, so when you eat less, be sure to eat a balanced diet.

3. A 22-year-old woman becomes pregnant. A fetal ultra- sound examination at 13 weeks' gestation shows her uterus measures 7 × 4 × 3 cm. At delivery of a term infant, her uterus measures 34 × 18 × 12 cm. Which of the following cellular pro- cesses has contributed most to the increase in her uterine size? A Endometrial glandular hyperplasia B Myometrial fibroblast proliferation C Endometrial stromal hypertrophy D Myometrial smooth muscle hypertrophy E Vascular endothelial hyperplasia

3. D The increase in uterine size is primarily the result of an increase in the size of myometrial smooth muscle cells. The endometrium also increases in size, mainly via hyperplasia, but it remains as a thin lining to the muscular wall and does not contribute as much to the change in size. There is little stroma in myometrium and a greater proportion in endometrium, so stroma contributes a smaller percentage to the gain in size than muscle. The vessels are a minor but essential component in this increase in size, but not the largest component.

3. A 4-year-old child has had a high-volume diarrhea for the past 2 days. On examination she is dehydrated. A stool sample examined by serologic assay is positive for rotavirus. She is treated with intravenous fluids and recovers. Which of the following components is found on intestinal cells and recognizes double-stranded RNA of this virus to signal transcription factors that upregulate interferon production for viral elimination? A Caspase-1 B Complement receptor C Lectin D T cell receptor E Toll-like receptor

3. E Nonhuman microbial substances such as double- stranded RNA of viruses, bacterial DNA, and bacterial endotoxin, can be recognized by Toll-like receptors (TLRs) on human cells as part of an innate defense mechanism against infection. Caspase-1 is activated by an inflammasome complex of proteins responding to bacterial organisms, and pro- duces biologically active interleukin-1 (IL-1). Complement receptors on inflammatory cells recognize complement components that aid in triggering immune responses through co- stimulatory signals. Lectins found on cell surfaces can bind a variety of substances, such as fungal polysaccharides, that trigger cellular defenses. T cell receptors respond to peptide antigens to trigger a cell-mediated immune response.

30. A 9-year-old boy has had a chronic cough and fever for the past month. A chest radiograph shows enlargement of hilar lymph nodes and bilateral pulmonary nodular interstitial infiltrates. A sputum sample contains acid-fast bacilli. A transbronchial biopsy specimen shows granulomatous inflammation with epithelioid macrophages and Langhans giant cells. Which of the following mediators is most likely to contribute to giant cell formation? A Complement C3b B Interferon-γ C Interleukin-1 (IL-1) D Leukotriene B4 E Tumor necrosis factor (TNF)

30. B Interferon-γ is secreted by activated T cells and is an important mediator of granulomatous inflammation. It causes activation of macrophages and their transformation into epithelioid cells and then giant cells. Complement C3b acts as an opsonin in acute inflammatory reactions. Interleukin-1 (IL-1) can be secreted by macrophages to produce various effects, including fever, leukocyte adherence, fibroblast proliferation, and cytokine secretion. Leukotriene B4 induces chemotaxis in acute inflammatory processes. Tumor necrosis factor (TNF) can be secreted by activated macrophages and induces activation of lymphocytes and proliferation of fibroblasts, which are other elements of a granuloma.

30. A new drug is developed that binds to cellular microtubules. The function of the microtubules is diminished, so that mitotic spindle formation is inhibited. Which of the following is the most likely use for this drug? A Antimicrobial therapy B Chemotherapy C Pain management D Prevention of atherosclerosis E Weight reduction

30. B Microtubules are cytoskeletal components required for cell movement. Mitotic spindles are required for cell division, and if cancer cells cannot divide, then the neoplasm cannot grow. Antibiotics are directed at microorganisms that do not have microtubules. Pain is produced largely through release of mediators of inflammation. Atheroma formation is affected by endothelial damage and lipid accumulation, and though there is cellular proliferation, it occurs over many years. Weight reduction is accomplished primarily via atrophy of adipocytes, not inhibition of cell proliferation.

31. A 46-year-old man has noted increasing abdominal size for the past 6 years. On physical examination his liver span is increased to 18 cm. An abdominal CT scan shows an enlarged liver with diffusely decreased attenuation. Laboratory findings include increased total serum cholesterol and triglyceride levels, increased prothrombin time, and a decreased serum albumin concentration. The representative microscopic appearance of his liver is shown in the figure. Which of the following activities most likely led to these findings? A Drinking beer B Ingesting aspirin C Injecting heroin D Playing basketball E Smoking cigarettes

31. A The appearance of lipid vacuoles in many of the hepatocytes is characteristic of fatty change (steatosis) of the liver. Abnormalities in lipoprotein metabolism can lead to steatosis. Alcohol is a hepatotoxin acting via increased acetaldehyde accumulation that promotes hepatic steatosis. Decreased serum albumin levels and increased prothrombin time suggest alcohol-induced hepatocyte damage. Aspirin has a significant effect on platelet function, but not on hepatocytes. Substance abuse with heroin produces few organ- specific pathologic findings. Exercise has little direct effect on hepatic function. Smoking directly damages lung tissue, but has no direct effect on the liver.

31. A 32-year-old woman has had a chronic cough with fever for the past month. On physical examination, her temperature is 37.5° C. A chest radiograph shows many small, ill-defined nodular opacities in all lung fields. A transbronchial biopsy specimen shows interstitial infiltrates with lymphocytes, plasma cells, and epithelioid macrophages. Which of the following infectious agents is the most likely cause of this appearance? A Candida albicans B Cytomegalovirus C Enterobacter aerogenes D Mycobacterium tuberculosis E Plasmodium falciparum F Staphylococcus aureus

31. D These findings suggest a granulomatous inflammation, and tuberculosis is a common cause. Candida is often a commensal organism in the oropharyngeal region and rarely causes pneumonia in healthy (non-immunosuppressed) individuals. Viral infections tend to produce a mononuclear interstitial inflammatory cell response. Bacteria such as Enterobacter and Staphylococcus are more likely to produce acute inflammation. Plasmodium produces malaria, a parasitic infection without a significant degree of lung involvement.

32. A 69-year-old woman has had transient ischemic attacks for the past 3 months. On physical examination, she has an audible bruit on auscultation of the neck. A right carotid end- arterectomy is performed. The curetted atheromatous plaque has a grossly yellow-tan, firm appearance. Microscopically, which of the following materials can be found in abundance in the form of crystals within cleftlike spaces? A Cholesterol B Glycogen C Hemosiderin D Immunoglobulin E Lipofuscin

32. A Cholesterol is a form of lipid commonly deposited within atheromas in arterial walls, imparting a yellow color to these plaques and a glistening appearance if abundant. Direct damage to the atheroma can yield cholesterol emboli. Glycogen is a storage form of carbo- hydrate seen mainly in liver and muscle. Hemosiderin is a storage form of iron that appears in tissues of the mono- nuclear phagocyte system (e.g., marrow, liver, spleen), but can be widely deposited with hereditary hemochromatosis. Immunoglobulin occasionally may be seen as rounded globules in plasma cells (i.e., Russell bodies). Lipofuscin is a golden brown pigment that increases with aging in cell cytoplasm, mainly in cardiac myocytes and in hepatocytes.

32. One month after an appendectomy, a 25-year-old woman palpates a small nodule beneath the skin at the site of the healed right lower quadrant sutured incision. The nodule is excised, and microscopic examination shows macrophages, collagen deposition, small lymphocytes, and multinucleated giant cells. Polarizable, refractile material is seen in the nodule. Which of the following complications of the surgery best accounts for these findings? A Abscess formation B Chronic inflammation C Exuberant granulation tissue D Granuloma formation E Healing by second intention

32. D The polarizable material is the suture, and a multinucleated giant cell reaction, typically with foreign body giant cells, is characteristic of a granulomatous reaction to foreign material. Granulation tissue may form a nodular appearance, and begins to appear 3 to 5 days following injury, but is unlikely to persist for a month. Chronic inflammation alone is unlikely to produce a localized nodule with giant cells. Edema refers to accumulation of fluid in the interstitial space. It does not produce a cellular nodule. If a large, gaping wound is not closed by sutures, it can granulate it and myofibroblastic contraction eventually helps close the wound by second intention.

33. A 43-year-old man has had a cough and fever for the past 2 months. A chest CT scan shows the findings in the fig- ure (A). A transbronchial lung biopsy is performed, yielding a specimen with the microscopic appearance shown in the figure (B). Which of the following chemical mediators is most important in the pathogenesis of this lesion? A Bradykinin B Complement C5a C Interferon-γ D Nitric oxide E Prostaglandins

33. C Figure A shows diffuse reticulonodular pulmonary densities, and Figure B shows noncaseating granulomas with many epithelioid cells and two prominent large Langhans giant cells. If special stains and/or cultures for organisms (usually mycobacteria or fungi) are negative, then this is likely sarcoidosis. Macrophage stimulation and transformation to epithelioid cells and giant cells are characteristic of granuloma formation. Interferon-γ promotes the formation of epithelioid cells and giant cells. Bradykinin is released in acute inflammatory responses and results in pain. Complement C5a is chemotactic for neutrophils. Although occasional neutrophils are seen in granulomas, neutrophils do not form a major component of granulomatous inflammation. Macrophages can release nitric oxide to destroy other cells, but nitric oxide does not stimulate macrophages to form a granulomatous response. Prostaglandins are mainly involved in the causation of vasodilation and pain in acute inflammatory responses.

33. A 45-year-old woman has had worsening dyspnea for the past 5 years. A chest CT scan shows panlobular emphysema. Laboratory studies show a deficiency of α1-antitrypsin (AAT). Her AAT genotype is PiZZ. A liver biopsy specimen examined microscopically shows abundant PAS-positive globules with- in periportal hepatocytes. Which of the following molecular mechanisms is most likely responsible for this finding in her hepatocytes? A Decreased catabolism of AAT in lysosomes B Excessive hepatic synthesis of AAT C Impaired dissociation of AAT from chaperones D Inability to metabolize AAT in Kupffer cells E Retained misfolded AAT in endoplasmic reticulum

33. E Mutations in the AAT gene give rise to AAT molecules that cannot fold properly. In the PiZZ genotype, both alleles have the mutation. The partially folded molecules accumulate in hepatocyte endoplasmic reticulum and can- not be secreted. Impaired dissociation of the CFTR protein from chaperones causes many cases of cystic fibrosis. There is no abnormality in the synthesis, catabolism, or metabolism of AAT in patients with AAT deficiency. AAT is the major circulating alpha globulin that protects tissues such as lung from damaging proteases.

34. At autopsy, the heart of a 63-year-old man weighs only 250 g (normal 330 g) and has small right and left ventricles. The myocardium is firm, with a dark chocolate-brown color throughout. The coronary arteries show minimal atherosclerotic changes. An excessive amount of which of the following substances, shown in the figure, would most likely be found in the myocardial fibers of this heart? A Bilirubin B Glycogen C Hemosiderin D Lipofuscin E Melanin

34. D Lipofuscin is a "wear-and-tear" pigment that in- creases with aging, particularly in liver and myocardium. This granular golden brown pigment seen adjacent to the myocyte nucleus in the figure has minimal effect on cellular function in most cases. Rarely, there is marked lipofuscin deposition in a small heart, a so-called brown atrophy. Bilirubin, another breakdown product of hemoglobin, imparts a yellow appearance (icterus) to tissues. Hemosiderin is the breakdown product of hemoglobin that contains the iron. Hearts with excessive iron deposition tend to be large. Glycogen is increased in some inherited enzyme disorders, and when the heart is involved, heart size increases. Melanin pigment is responsible for skin tone: the more melanin, the darker the skin.

34. An 8-year-old girl has had difficulty swallowing for the past day. On examination, her pharynx is swollen and erythematous with an overlying yellow exudate. Laboratory studies show neutrophilia. Streptococcus pyogenes (group A streptococcus) is cultured from her pharynx. Which of the following substances is most likely to increase in response to pyrogens released by this organism? A Hageman factor B Immunoglobulin E C Interleukin-12 (IL-12) D Nitric oxide E Prostaglandins

34. E The findings here are those of strep throat with acute inflammation. Bacterial organisms often lead to fever accompanying infection through release of exogenous pyrogens that induce inflammatory cells to release endogenous pyro- gens such as tumor necrosis factor (TNF) and interleukin-1 (IL-1). The pyrogens stimulate prostaglandin synthesis in the hypothalamus to "reset the thermostat," so that fever occurs as a sign of the acute inflammatory response. Hageman factor initiates the coagulation cascade. Immunoglobulin E is often increased in response to inflammatory responses with allergens and with invasive parasites. Interleukin-12 (IL-12) released by macrophages stimulates T-cell responses. Nitric oxide generated in endothelium leads to vasodilation, where- as nitric oxide produced in macrophages aids in microbial killing.

35. A 69-year-old woman has had a chronic cough for the past year. A chest radiograph shows a 6-cm mass in the left lung. A needle biopsy specimen of the mass shows carcinoma. A pneumonectomy is performed, and examination of the hilar lymph nodes reveals a uniform, dark black cut surface. Which of the following factors most likely accounts for the appearance of these lymph nodes? A Aging effects B Bleeding disorder C Cigarette smoking D Liver failure E Multiple metastases

35. C Lung and hilar lymph nodes accumulate anthracotic pigmentation when carbon pigment is inhaled from polluted air. The tar in cigarette smoke is a major source of such car- bonaceous pigment. Older individuals generally have more anthracotic pigment, but this is not inevitable with aging— individuals living in rural areas with good environmental air quality have less pigment. Resolution of hemorrhage can produce hemosiderin pigmentation, which imparts a brown color to tissues. Hepatic failure may result in jaundice, characterized by a yellow color in tissues. Metastases are mass lesions that impart a tan-to-white appearance to tissues.

35. A 41-year-old man has had a severe headache for the past 2 days. On examination, his temperature is 39.2° C. A lumbar puncture is performed, and the cerebrospinal fluid obtained has a WBC count of 910/mm3 with 94% neutrophils and 6% lymphocytes. Which of the following substances is the most likely mediator for the fever observed in this man? A Bradykinin B Histamine C Leukotriene B4 D Nitric oxide E Tumor necrosis factor (TNF)

35. E Fever is produced by various inflammatory mediators, but the major cytokines that produce fever are interleukin-1 (IL-1) and tumor necrosis factor (TNF), which are produced by macrophages and other cell types. IL-1 and TNF can have autocrine, paracrine, and endocrine effects. They mediate the acute phase responses, such as fever, nausea, and neutrophil release from bone marrow. Brady- kinin, generated from the kinin system on surface contact of Hageman factor with collagen and basement membrane from vascular injury, promotes vascular permeability, smooth muscle contraction, and pain. Histamine released from mast cells is a potent vasodilator, increasing vascular permeability. Leukotriene B4, generated in the lipoxygenase pathway of arachidonic acid metabolism, is a potent neutrophil chemotactic factor. Nitric oxide generated by macro- phages aids in destruction of microorganisms; nitric oxide released from endothelium mediates vasodilation and inhibits platelet activation.

36. A 43-year-old man with a ventricular septal defect has had a cough and fever for the past 2 days. On examination, he has a temperature of 37.6° C and a cardiac murmur. A blood culture grows Streptococcus, viridans group. His erythrocyte sedimentation rate (ESR) is increased. Microbial cells are opsonized and cleared. Which of the following chemical mediators is most important in producing these findings? A Bradykinin B C-reactive protein C Interferon-γ D Nitric oxide E Prostaglandin F Tumor necrosis factor (TNF)

36. B This acute inflammatory process leads to production of acute-phase reactants, such as C-reactive protein (CRP), fibrinogen, and serum amyloid A (SAA) protein. These proteins, particularly fibrinogen, and immunoglobulins increase RBC rouleaux formation to increase the erythrocyte sedimentation rate (ESR), which is a nonspecific indicator of inflammation. CRP production is upregulated by interleukin-6 (IL-6), whereas fibrinogen and SAA are upregulated mainly by tumor necrosis factor (TNF) and interleukin-1 (IL-1). Interferon-γ is a potent stimulator of macrophages. Nitric oxide can induce vasodilation or can assist in microbial killing within macro- phages. Prostaglandins are vasodilators.

36. A 22-year-old woman from Albania has a congenital anemia requiring multiple transfusions of RBCs for many years. On physical examination, her skin has a bronze color. Liver function tests show reduced serum albumin. Which of the following findings would most likely appear in a liver biopsy specimen? A Amyloid in portal triads B Bilirubin in canaliculi C Glycogen in hepatocytes D Hemosiderin in hepatocytes E Steatosis in hepatocytes

36. D Each unit of blood contains about 250 mg of iron. The body has no mechanism for getting rid of excess iron. About 10 to 20 mg of iron per day is lost with normal desquamation of epithelia; menstruating women lose slightly more. Any excess iron becomes storage iron, or hemosiderin. Over time, hemosiderosis involves more and more tissues of the body, particularly the liver, but also skin. Initially, hemosiderin deposits are found in Kupffer cells and other mono- nuclear phagocytes in the bone marrow, spleen, and lymph nodes. With great excess of iron, liver cells also accumulate iron. Amyloid is an abnormal protein derived from a variety of precursors, such as immunoglobulin light chains. Bilirubin, a breakdown product of blood, can be excreted in the bile so that a person does not become jaundiced. Glycogen storage diseases are inherited and present in childhood. Steatosis usually occurs with ingestion of hepatotoxins, such as alcohol.

39. A 54-year-old man undergoes laparoscopic hernia re- pair. In spite of the small size of the incisions, he has poor wound healing. Further history reveals that his usual diet has poor nutritional value and is deficient in vitamin C. Synthesis of which of the following extracellular matrix components is most affected by this deficiency? A Collagen B Elastin C Fibronectin D Integrin E Laminin

39. A Vitamin C deficiency leads to scurvy, with reduced lysyl oxidase enzyme activity that helps cross-link fibrillar collagens to provide tensile strength. Though elastin is a fibrillar protein, it tends to regenerate poorly in scar tissue, even with the best of nutrition, explaining why a scar does not stretch like the skin around it. The other listed choices are glycoproteins that have an adhesive quality and are not vitamin C dependent.

37. A 72-year-old man died suddenly from congestive heart failure. At autopsy, his heart weighed 580 g (normal 330 g) and showed marked left ventricular hypertrophy and minimal coronary arterial atherosclerosis. A serum chemistry panel or- dered before death showed no abnormalities. Which of the fol- lowing pathologic processes best accounts for the appearance of the aortic valve seen in the figure? A Amyloidosis B Dystrophic calcification C Hemosiderosis D Hyaline change E Lipofuscin deposition

37. B The valve is stenotic because of nodular deposits of calcium. The process is "dystrophic" because calcium deposition occurs in damaged tissues. The damage in this patient is a result of excessive wear and tear with aging. Amyloid deposition in the heart typically occurs within the myocardium and the vessels. Hereditary hemochromatosis is a genetic defect in iron absorption that results in extensive myocardial iron deposition (hemosiderosis). Hyaline change is a descriptive term used by histologists to describe protein deposits that are glassy and pale pink. The amount of lipofuscin increases within myocardial fibers (not valves) with aging.

37. In an experiment, a group of test animals is infected with viral hepatitis. Two months later, complete recovery of the normal liver architecture is observed microscopically. A control test group is infected with bacterial organisms, and after the same period of time, fibrous scars from resolving hepatic abscesses are seen microscopically. Which of the following factors best explains the different outcomes for the two test groups? A Extent of damage to the biliary ducts B Extent of the hepatocyte injury C Injury to the connective tissue framework D Location of the lesion within the liver E Nature of the injurious etiologic agent

37. C Hepatocytes are stable cells with an extensive ability to regenerate. The ability to restore normal architecture of an organ such as the liver depends on the viability of the supporting connective tissue framework. If the connective tissue cells are not injured, hepatocyte regeneration can restore normal liver architecture. This regeneration occurs in many cases of viral hepatitis. A liver abscess associated with liquefactive necrosis of hepatocytes and the supporting connective tissue heals by scarring. The other options listed may explain the amount of liver injury, but not the nature of the response.

38. A 51-year-old woman tests positive for hepatitis A anti- body. Her serum AST level is 275 U/L, and ALT is 310 U/L. One month later, these enzyme levels have returned to normal. Which phase of the cell cycle best describes the hepatocytes 1 month after her infection? A G0 B G1 C S D G2 EM

38. A Hepatocytes are quiescent (stable) cells that can reenter the cell cycle and proliferate in response to hepatic injury, enabling the liver to regenerate partially. Acute hepatitis results in hepatocyte necrosis, marked by elevations in AST and ALT. After the acute process has ended, cells return to the G0 phase, and the liver becomes quiescent again.

38. A 70-year-old man with hypercalcemia died suddenly. At autopsy, microscopic examination showed noncrystalline amorphous deposits of calcium salts in gastric mucosa, renal interstitium, and alveolar walls of lungs. Which of the following underlying conditions would most likely explain these findings? A. Chronic active hepatitis B. Diffuse parathyroid hyperplasia C. Disseminated tuberculosis D. Generalized atherosclerosis E. Normal aging process F. Pulmonary emphysema

38. B The microscopic findings suggest metastatic calcification, with deposition of calcium salts in tissues that have physiologic mechanisms for losing acid, creating an internal alkaline environment that favors calcium precipitation. Hypercalcemia can have a variety of causes, including primary and secondary hyperparathyroidism, bone destruction secondary to metastases, paraneoplastic syndromes, and, less commonly, vitamin D toxicity or sarcoidosis. Chronic renal disease re- duces phosphate excretion by the kidney, resulting in an in- crease in serum phosphate. Because the solubility product of calcium and phosphorus must be maintained, the serum calcium is depressed, triggering increased parathyroid hormone output to increase the calcium level, which promotes calcium deposition. Chronic hepatitis leads to hyperbilirubinemia and jaundice. The granulomas of tuberculosis have caseous necrosis with dystrophic calcification. Another form of dystrophic calcification occurs when atherosclerotic lesions calcify. Dystrophic calcification is seen more often in the elderly, but it is the result of a lifetime of pathologic changes, not aging itself. Pulmonary emphysema can lead to respiratory acidosis that is compensated by metabolic alkalosis, with the result that the serum calcium level remains relatively unchanged.

39. An experiment analyzes cells for enzyme activity associated with sustained cellular proliferation. Which of the following cells is most likely to have the highest telomerase activity? A Endothelial cells B Erythrocytes C Germ cells D Neurons E Neutrophils

39. C Germ cells have the highest telomerase activity, and the telomere length can be stabilized in these cells. This al- lows testicular germ cells to retain the ability to divide throughout life. Normal somatic cells have no telomerase activity, and telomeres progressively shorten with each cell division until growth arrest occurs. Erythrocytes do not even have a nucleus.

4. A 20-year-old woman breastfeeds her infant. On examination, her breasts are slightly increased in size. Milk can be expressed from both nipples. Which of the following processes that occurred in her breasts during pregnancy enables her to breastfeed the infant? A Ductal metaplasia B Epithelial dysplasia C Intracellular lipid deposition D Lobular hyperplasia E Stromal hypertrophy

4. D Breast lobules have an increased number of cells under hormonal influence (mainly progesterone) to provide for normal lactation. Ductal metaplasia in the breast is a pathologic process. Epithelial dysplasia denotes disordered growth and maturation of epithelial cells that may progress to cancer. Accumulation of fat within cells is a common manifestation of sublethal cell injury or, uncommonly, of inborn errors in fat metabolism. The breast stroma plays no role in lactation and may increase with pathologic processes.

4. A 40-year-old woman has had chronic congestive heart failure for the past 3 years. In the past 2 months, she developed a cough productive of rust-colored sputum. A sputum cytology specimen now shows numerous hemosiderin-laden macrophages. Which of the following subcellular structures in these macrophages is most important for the accumulation of this pigment? A. Chromosome B. Endoplasmic reticulum C. Golgi apparatus D. Lysosome E. Ribosome

4. D Heterophagocytosis by macrophages requires that endocytosed vacuoles fuse with lysosomes to degrade the engulfed material. With congestive heart failure, extravasation of RBCs into alveoli occurs, and pulmonary macrophages must phagocytose the RBCs, breaking down the hemoglobin and recycling the iron by hemosiderin formation. The other listed options are components that play a role in cell synthetic functions.

4. A 72-year-old man with severe emphysema has had worsening right ventricular failure for the past 5 years. For the past 4 days, he has had fever and increasing dyspnea. A chest radiograph shows an accumulation of fluid in the pleural spaces. Fluid obtained by thoracentesis has a specific gravity of 1.030 and contains degenerating neutrophils. The most likely cause of this fluid accumulation is due to changes in which of the following? A Colloid osmotic pressure B Leukocytic diapedesis C Lymphatic pressure D Renal sodium retention E Vascular permeability

4. E The formation of an exudate containing a significant amount of protein and cells depends on the "leakiness" of blood vessels, principally venules. When exudation has occurred, the protein content of the extravascular space in- creases, and extravascular colloid osmotic pressure increases, causing extracellular fluid accumulation. Leukocytosis alone is insufficient for exudation because the leukocytes must be driven to emigrate from the vessels by chemotactic factors. The lymphatics scavenge exuded proteinaceous fluid and reduce the amount of extravascular and extracellular fluid. Sodium and water retention helps drive transudation of fluid.

40. A study of aging shows that senescent cells have accumulated damage from toxic byproducts of metabolism. There is increased intracellular lipofuscin deposition. Prolonged ingestion of which of the following substances is most likely to counteract this aging mechanism? A Antioxidants B Analgesics C Antimicrobials D Antineoplastic agents E Glucocorticoids

40. A Antioxidants may counteract the effects of reactive oxygen species (ROS) that may accumulate acutely and chronically within cells as a consequence of environmental insults and pathologic processes. Certainly, health food stores promote this concept with sales of products such as vitamin E. However, cellular damage is multifactorial, and proving that one compound has a significant effect is difficult. Analgesics ameliorate the perception of pain from cellular damage, but they do not prevent or diminish cell damage; they only mask it. Antimicrobials may help the body's own immune defenses against infectious agents and shorten and/or diminish tissue damage. However, long- term use of antimicrobials is discouraged because it may alter the body's own useful microbial flora, and it can pro- mote development of drug-resistant strains that pose a serious health risk for the general population. (As Mr. Spock noted, "The needs of the many outweigh the needs of the few.") Antineoplastic agents are given for malignancies and rarely have benefit for cancer prevention. Glucocorticoids provide short-term improvement in well-being, but when used for longer periods, they have deleterious effects.

40. In an experiment, glass beads are embolized into the coronary arteries of rats, resulting in myocardial injury. After 7 days, sections of the myocardium are studied using light microscopy. The microscopic appearance of one of these sections is shown in the figure. Which of the following mediators is most likely being expressed to produce this appearance? A Epidermal growth factor B Interleukin-2 (IL-2) C Leukotriene B4 D Thromboxane A2 E Tumor necrosis factor (TNF) F Vascular endothelial growth factor

40. F The figure shows a subacute infarction with granulation tissue formation containing numerous capillaries stimulated by vascular endothelial growth factor, representing a healing response. Epidermal growth factor aids in reepithelialization of a surface wound. Interleukin-2 (IL-2) mediates lymphocyte activation. Leukotriene B4 mediates vasoconstriction and bronchoconstriction. Thromboxane A2 aids vasoconstriction and platelet aggregation. Tumor necrosis factor (TNF) induces endothelial activation and many responses that occur secondary to inflammation, including fever, loss of appetite, sleep disturbances, hypotension, and increased corticosteroid production.

41. A 20-year-old woman undergoes cesarean section to deliver a term infant, and the lower abdominal incision is sutured. The sutures are removed 1 week later. Which of the following statements best describes the wound site at the time of suture removal? A Collagen degradation exceeds synthesis B Granulation tissue is still present C No more wound strength will be gained D Type IV collagen predominates E Wound strength is 80% of normal tissue

41. A At 1 week, wound healing is incomplete, and granulation tissue is still present. More collagen is synthesized in the following weeks. Wound strength peaks at about 80% by 3 months. Type IV collagen is found in basement membranes.

42. A 24-year-old man with acute appendicitis undergoes surgical removal of the inflamed appendix. The incision site is sutured. A trichrome-stained section representative of the site with blue appearing collagen is shown in the figure. How long after the surgery would this appearance most likely be seen? A 1 day B 2 to 3 days C 4 to 5 days D 2 weeks E 1 month

42. E The figure shows dense collagen with some remaining dilated blood vessels, typical of the final phase of wound healing, which is extensive by the end of the first month. On day 1, the wound is filled only with fibrin and inflammatory cells. Macrophages and granulation tissue are seen 2 to 3 days postoperatively. Neovascularization is most prominent by days 4 and 5. By week 2, collagen is prominent, and fewer vessels and inflammatory cells are seen.

43. A 40-year-old man underwent laparotomy for a perforated sigmoid colon diverticulum. A wound infection complicated the postoperative course, and surgical wound dehiscence occurred. Primary closure was no longer possible, and the wound "granulated in." Six weeks later, the wound is only 10% of its original size. Which of the following processes best accounts for the observed decrease in wound size over the past 6 weeks? A Elaboration of adhesive glycoproteins B Increase in synthesis of collagen C Inhibition of metalloproteinases D Myofibroblast contraction E Resolution of subcutaneous edema

43. D Wound contraction is a characteristic feature of healing by second intention that occurs in larger wounds. Collagen synthesis helps fill the defect, but does not contract it. Adhesive glycoproteins such as fibronectin help to maintain a cellular scaffolding for growth and repair, but they do not contract. The inhibition of metalloproteinases leads to decreased degradation of collagen and impaired connective tissue remodeling in wound repair. Edema diminishes over time, but this does not result in much contraction.

44. In an experiment involving observations on wound healing, researchers noted that intracytoplasmic cytoskeletal elements, including actin, interact with the extracellular matrix to promote cell attachment and migration in wound healing. Which of the following substances is most likely responsible for such interaction between the cytoskeleton and the extracellular matrix? A Epidermal growth factor B Fibronectin C Integrin D Platelet-derived growth factor E Type IV collagen F Vascular endothelial growth factor

44. C Integrins interact with the extracellular matrix proteins (e.g., fibronectin). Engagement of integrins by extracellular matrix proteins leads to the formation of focal adhesions where integrins link to intracellular cytoskeletal elements such as actin. These interactions lead to intracellular signals that modulate cell growth, differentiation, and migration during wound healing. Epidermal growth factor stimulates epithelial cell and fibroblast proliferation. Platelet-derived growth factor (PDGF) can be produced by endothelium, macrophages, smooth muscle cells, and platelets; PDGF mediates migration and proliferation of fibroblasts and smooth muscle cells and migration of monocytes. Type IV collagen is found in basement membranes on which cells are anchored. Vascular endothelial growth factor promotes angiogenesis (capillary proliferation) through endothelial cell proliferation and migration in a healing response.

45. A 23-year-old woman receiving corticosteroid therapy for an autoimmune disease has an abscess on her upper outer right arm. She undergoes minor surgery to incise and drain the abscess, but the wound heals poorly over the next month. Which of the following aspects of wound healing is most likely to be deficient in this patient? A Collagen deposition B Elaboration of VEGF C Neutrophil infiltration D Reepithelialization E Serine proteinase production

45. A Glucocorticoids inhibit wound healing by impairing collagen synthesis. This is a desirable side effect if the amount of scarring is to be reduced, but it results in the delayed healing of surgical wounds. Angiogenesis driven by vascular endothelial growth factor (VEGF) is not significantly affected by corticosteroids. Neutrophil infiltration is not prevented by glucocorticoids. Reepithelialization, in part driven by epidermal growth factor, is not affected by corticosteroid therapy. Serine proteinases are important in wound remodeling.

5. A 35-year-old man has had increasing dyspnea for the past 24 hours. A chest radiograph shows large, bilateral pleural effusions. Thoracentesis yields 500 mL of slightly cloudy yellow fluid from the right pleural cavity. Cytologic examination of the fluid shows many neutrophils, but no lymphocytes or RBCs. Which of the following mechanisms contributes most to the pleural fluid accumulation? A Arteriolar vasoconstriction B Endothelial contraction C Inhibition of platelet adherence D Lymphatic obstruction E Neutrophil release of lysosomes

5. B Exudation of fluid from venules and capillaries is a key component of the acute inflammatory process. Several mechanisms of increased vascular permeability have been proposed, including formation of interendothelial gaps by contraction of endothelium. This contraction can be caused by mediators such as histamine and leukotrienes. The vessels then become more "leaky," and the fluid leaves the intravascular space to accumulate extravascularly, forming effusions in body cavities or edema within tissues. Arteriolar vasoconstriction is a transient response to injury that helps diminish blood loss. Platelets adhere to damaged endothelium and promote hemostasis. Lymphatic obstruction results in the ac- cumulation of protein-rich lymph and lymphocytes, producing a chylous effusion within a body cavity. After neutrophils reach the site of tissue injury outside of the vascular space, they release lysosomal enzymes that promote liquefaction.

5. A 16-year-old boy sustained blunt trauma to his abdo- men when he struck a bridge abutment at high speed while driving a motor vehicle. Peritoneal lavage shows a hemoperi- toneum, and at laparotomy, a small portion of the left lobe of the injured liver is removed. Two months later, a CT scan of the abdomen shows that the liver has nearly regained its size before the injury. Which of the following processes best ex- plains this CT scan finding? A Apoptosis B Dysplasia C Hyperplasia D Hydropic change E Steatosis

5. C The liver is one of the few organs in the human body that can partially regenerate. This is a form of compensatory hyperplasia. The stimuli to hepatocyte mitotic activity cease when the liver has attained its normal size. Hepatocytes can reenter the cell cycle and proliferate to regenerate the liver; they do not just hypertrophy (increase in size). Apoptosis is single cell death and frequently occurs with viral hepatitis. Dysplasia is disordered epithelial cell growth that can be premalignant. Hydropic change, or cell swelling, does not produce regeneration. Steatosis (fatty change) can lead to hepatomegaly, but not as a regenerative process. It is the result of toxic/metabolic hepatocyte injury.

5. An experiment is conducted in which cells in tissue culture are subjected to high levels of ultraviolet radiant energy. Electron microscopy shows cellular damage in the form of increased cytosolic aggregates of denatured proteins. In situ hybridization reveals that protein components in these aggregates also are found in proteasomes. Which of the following substances most likely binds to the denatured proteins, targeting them for catabolism by cytosolic proteasomes? A. Adenosine monophosphate B. Calcium C. Caspase D. Granzyme B E. Hydrogen peroxide F. Ubiquitin

5. F Heat-shock proteins provide for a variety of cellular "housekeeping" activities, including recycling and restoration of damaged proteins and removal of denatured proteins. Ubiquitin targets denatured proteins and facilitates their binding to proteasomes, which then break down the proteins to peptides. ADP increases when ATP is depleted, helping to drive anaerobic glycolysis. Cytosolic calcium lev- els may increase with cell injury that depletes ATP; the calcium activates phospholipases, endonucleases, and proteases, which damage the cell membranes, structural proteins, and mitochondria. Caspases are enzymes that facilitate apoptosis. Granzyme B is released from cytotoxic T lymphocytes and triggers apoptosis. Hydrogen peroxide is one of the activated oxygen species generated under conditions of cellular ischemia, producing nonspecific damage to cellular structures, particularly membranes.

6. At the site of a surgical incision, endothelial cells elaborate vascular endothelial growth factor. There is sprouting with migration of endothelial cells into the wound to establish new capillaries. Which of the following intracellular proteins is most important in facilitating movement of endothelial cells? A. Actin B. Cytokeratin C. Desmin D. Lamin E. Myosin

6. A Actin is a microfilament involved with cell movement. The other possibilities listed in B to D are intermediate filaments, which are larger than actin but smaller than myosin (a thick filament interdigitating with actin, required for muscle movement). Cytokeratins form cytoskeletal elements of epithelial cells. Desmin forms the scaffold in muscle cells on which actin and myosin contract. Lamin is associated with the nuclear membrane.

6. A 71-year-old man has had difficulty with urination, in- cluding hesitancy and increased frequency, for the past 5 years. A digital rectal examination reveals that his prostate gland is palpably enlarged to twice normal size. A transurethral resection of the prostate is performed, and the microscopic appearance of the prostate "chips" obtained is that of nodules of glands with intervening stroma. Which of the following pathologic processes has most likely occurred in his prostate? A Apoptosis B Dysplasia C Fatty change D Hyperplasia E Hypertrophy F Metaplasia

6. D Nodular prostatic hyperplasia (also known as benign prostatic hyperplasia [BPH]) is a common condition in older men that results from proliferation of both prostatic glands and stroma. The prostate becomes more sensitive to androgenic stimulation with age. This is an example of pathologic hyperplasia. Apoptosis results in a loss of, not an increase in, cells. Dysplasia refers to disordered epithelial cell growth and maturation. Fatty change in hepatocytes may produce hepatomegaly. Although BPH is often called "benign prostatic hypertrophy," this term is technically incorrect; it is the number of glands and stromal cells that is increased, rather than the size of existing cells. A change in the glandular epithelium to squamous epithelium around a prostatic infarct would be an example of metaplasia.

6. A 6-year-old child has a history of recurrent infections with pyogenic bacteria, including Staphylococcus aureus and Streptococcus pneumoniae. The infections are accompanied by a neutrophilic leukocytosis. Microscopic examination of a biopsy specimen obtained from an area of soft tissue necrosis shows microbial organisms, but very few neutrophils. An analysis of neutrophil function shows a defect in rolling. This child's increased susceptibility to infection is most likely caused by a defect involving which of the following molecules? A Complement C3b B Integrins C Leukotriene B4 D NADPH oxidase E Selectins

6. E Leukocyte rolling is the first step in transmigration of neutrophils from the vasculature to the tissues. Rolling depends on interaction between selectins (P-selectin and E-selectin on endothelial cells, and L-selectin on neutrophils) and their sialylated ligands (e.g., sialylated Lewis X). Integrins are involved in the next step of transmigration, during which there is firm adhesion between neutrophils and endothelial cells. Complement C3b acts as an opsonin to facilitate phagocytosis. Leukotriene B4 is a chemotactic agent. NADPH oxidase is involved in phagocytic cell microbicidal activity.

7. In an experiment, bacteria are introduced into a per- fused tissue preparation. Leukocytes leave the vasculature and migrate to the site of bacterial inoculation. The movement of these leukocytes is most likely to be mediated by which of the following substances? A Bradykinin B Chemokines C Complement C3a D Histamine E Prostaglandins

7. B Chemokines include many molecules that are chemo- tactic for neutrophils, eosinophils, lymphocytes, monocytes, and basophils. Bradykinin causes pain and increased vascular permeability. Complement C3a causes increased vascular permeability by releasing histamine from mast cells. Histamine causes vascular leakage. Prostaglandins have multiple actions, but they do not cause chemotaxis.

7. A 29-year-old man sustains a left femoral fracture in a motorcycle accident. His leg is placed in a plaster cast. After his left leg has been immobilized for 6 weeks, the diameter of the left calf has decreased in size. This change in size is most likely to result from which of the following alterations in his calf muscles? A Aplasia B Atrophy C Dystrophy D Hyalinosis E Hypoplasia

7. B Reduced workload causes cell to shrink through loss of cell substance, a process called atrophy. The cells are still present, just smaller. Aplasia refers to lack of embryonic development; hypoplasia describes poor or subnormal development of tissues. Dystrophy of muscles refers to inherited disorders of skeletal muscles that lead to muscle fiber destruction, weakness, and wasting. Hyaline change (hyalinosis) refers to a nonspecific, pink, glassy eosinophilic appearance of cells.

7. In an experiment, release of epidermal growth factor into an area of denuded skin causes mitogenic stimulation of the skin epithelial cells. Which of the following proteins is most likely to be involved in transducing the mitogenic signal from the epidermal cell membrane to the nucleus? A. Cyclic AMP B. Cyclin D C. Cyclin-dependent kinase D. G proteins E. RAS proteins

7. E RAS proteins transduce signals from growth factor re- ceptors, such as epidermal growth factor, that have intrinsic tyrosine kinase activity. G proteins perform a similar function for G protein-linked, seven-transmembrane receptors. Cyclic AMP is an effector in the G protein signaling pathway. Cyclins and cyclin-dependent kinases regulate the cell cycle in the nucleus.

8. A 34-year-old obese woman has experienced heartburn from gastric reflux for the past 5 years after eating large meals. She undergoes upper gastrointestinal endoscopy, and a biopsy specimen of the distal esophagus is obtained. Which of the following microscopic changes, seen in the figure, has most likely occurred? A Columnar metaplasia B Goblet cell hyperplasia C Lamina propria atrophy D Squamous dysplasia E Mucosal hypertrophy

8. A Inflammation from reflux of gastric acid has resulted in replacement of normal esophageal squamous epithelium by intestinal-type columnar epithelium with goblet cells. Such conversion of one adult cell type to another cell type is called metaplasia, and it occurs when stimuli reprogram stem cells. Goblet cells are not normal constituents of the esophageal mucosa, and they are a minor part of this meta- plastic process. The lamina propria has some inflammatory cells, but it does not atrophy. The squamous epithelium does not become dysplastic from acid reflux, but the columnar metaplasia may progress to dysplasia, not seen here, if the abnormal stimuli continue. These cells are not significantly increased in size (hypertrophic).

8. A 12-month-old boy with a 6-month history of repeated infections has had a fever and cough for the past 3 days. A Gram stain of sputum shows many gram-positive cocci in chains. CBC shows neutrophilia. Laboratory studies show that the patient's neutrophils phagocytose and kill organisms promptly in the presence of normal human serum, but not in his own serum. The neutrophils migrate normally in a chemotaxis assay. Which of the following is the most likely cause of this boy's increased susceptibility to infection? A Abnormality of selectin expression B Diminished opsonization C Defective neutrophil generation of hydrogen peroxide D Deficiency of integrins E Phagocytic cell microtubular protein defect

8. B This immunoglobulin deficiency prevents opsonization and phagocytosis of microbes. Deficiency of integrins and selectins, or a defect in microtubules, would prevent adhesion and locomotion of neutrophils. H2O2 production is part of the oxygen-dependent killing mechanism. This mechanism is intact in this patient because the neutrophils are able to kill bacteria when immunoglobulins in normal serum allow phagocytosis.

8. Various soluble mediators are added to a cell culture containing epidermal cells to determine which of the mediators might be useful for promoting epidermal cell growth. When epidermal growth factor (EGF) is added, it binds to epidermal cell surface receptors, with subsequent transcription factor translocation and DNA transcription. This effect in the epidermal cells is most likely to be mediated through which of the following intracellular pathways? A. Calcium ion channel B. Cyclic AMP C. Cyclin-dependent kinase D. JAK/STAT system E. Mitogen-activated protein (MAP) kinase

8. E The MAP kinase cascade is involved in signaling from activation via cell surface receptors for growth factors. This pathway is particularly important for signaling of EGF and fibroblast growth factor. Ligand binding, such as occurs with acetylcholine at a nerve-muscle junction, alters the conformation of ion channel receptors to allow flow of spe- cific ions such as calcium into the cell, changing the electric potential across the cell membrane. Cyclic AMP is a second messenger that is typically activated via ligand binding to receptors with seven transmembrane segments that associ- ate with GTP-hydrolyzing proteins; chemokine receptors function in this fashion. Cyclin-dependent kinases act within the nucleus. JAK/STAT pathways typically are recruited by cytokine receptors.

9. An experiment involves factors controlling wound healing. Skin ulcerations are observed, and the factors involved in the healing process are analyzed. Which of the following factors is most likely to be effective in promoting angiogenesis? A. Basic fibroblast growth factor B. Endostatin C. Epidermal growth factor D. Interleukin-1 E. Platelet-derived growth factor

9. A Basic fibroblast growth factor is a potent inducer of angiogenesis. It can participate in all steps of angiogenesis. Endostatin is an inhibitor of angiogenesis. Epidermal growth factor and interleukin-1 have no significant angiogenic activity. Platelet-derived growth factor plays a role in vascular remodeling.

9. An 11-year-old girl becomes infected with hepatitis A and experiences mild nausea for 1 week. On physical examination, she has minimal right upper quadrant tenderness and scleral icterus. Laboratory findings include a serum AST of 68 U/L, ALT of 75 U/L, and total bilirubin of 5.1 mg/dL. Her laboratory findings most likely result from which of the following changes in her hepatocytes? A Cell membrane defects B Lysosomal autophagy C Mitochondrial swelling D Nuclear chromatin clumping E Ribosomal dispersion

9. A Irreversible cell injury is associated with loss of membrane integrity. This allows intracellular enzymes such as AST and ALT to leak into the serum. All other morphologic changes listed are associated with reversible cell injury, in which the cell membrane remains intact and the cells do not die.

9. A 5-year-old child has a history of recurrent bacterial infections, including pneumonia and otitis media. Analysis of leukocytes collected from the peripheral blood shows a deficiency in myeloperoxidase. A reduction in which of the following processes is the most likely cause of this child's in- creased susceptibility to infections? A Hydrogen peroxide (H2O2) elaboration B Hydroxy-halide radical (HOCl-) formation C Failure of migration resulting from complement deficiency D Phagocytic cell oxygen consumption E Prostaglandin production

9. B Myeloperoxidase is present in the azurophilic granules of neutrophils. It converts H2O2 into HOCl-, a powerful oxidant and antimicrobial agent. Degranulation occurs as phagolysosomes are formed with engulfed bacteria in phagocytic vacuoles within the neutrophil cytoplasm. Oxygen consumption with an oxidative or respiratory burst after phagocytosis is aided by glucose oxidation and activation of neutrophil NADPH oxidase, resulting in generation of superoxide that is converted by spontaneous dismutation to H2O2. In contrast, prostaglandin production depends on a functioning cyclooxygenase pathway of arachidonic acid metabolism.


Set pelajaran terkait

Sonidos en Contexto 24 [ɾ] [r] [h͡r] [r̥] [ɹ̝] [ɹ̝̊]

View Set

4th Grade Language Arts Module 4 Exam Study Guide

View Set

Ex Questions (Ch09,10,14,16,17&21) for Final Exam

View Set

Chapter 6:Assessing Mental Status Including Risk for Substance Abuse

View Set

French Diction Forbidden Liaison/Elision

View Set

Chapter 9 - Small Business Marketing (Product and Pricing Strategies)

View Set

Quiz: Other patterns of inheritance

View Set

Drugs and Behavior Chapter 6: Hallucinogens

View Set

Competitors, Collaborators, and Customers

View Set